一橋大2016:第2問


\(\theta\) を実数とし, 数列 \(\{ a_n \}\) を \[ a_1 = 1 , \quad a_2 = \cos \theta , \quad a _{n+2} = \dfrac{3}{2} a _{n+1} -a_n \ ( n = 1, 2, 3, \cdots ) \] により定める. すべての \(n\) について \(a_n = \cos (n-1) \theta\) が成り立つとき, \(\cos \theta\) を求めよ.


【 解 答 】

条件より \[\begin{align} \cos (n+1) \theta & = \dfrac{3}{2} \cos n \theta -\cos (n-1) \theta \\ 2 \cos n \theta \cos \theta & = \dfrac{3}{2} \cos n \theta \\ \cos n \theta \left( \cos \theta -\dfrac{3}{4} \right) & = 0 \\ \text{∴} \quad \cos n \theta = 0 & , \ \cos \theta = \dfrac{3}{4} \end{align}\] \(\cos n \theta = 0\) のときを考えると, \(a_2 = a_3 = 0\) となるが \[ a_1 = \dfrac{3}{2} a_2 -a_3 = 0 \neq 1 \] で, 矛盾が生じる.
よって \[ \cos \theta = \underline{\dfrac{3}{4}} \]

コメントを残す

このサイトはスパムを低減するために Akismet を使っています。コメントデータの処理方法の詳細はこちらをご覧ください